How to Find dz/dt in a Derivative with 3 Variables?

Click For Summary
To find dz/dt for the equation 1/z = 1/x + 1/y, implicit differentiation with respect to time t is necessary. The relationship between x and y is nonlinear, so dz/dt cannot simply be calculated as dx/dt + dy/dt. Instead, the correct approach involves using the chain rule, leading to the equation -1/z² * dz/dt = -1/x² * dx/dt - 1/y² * dy/dt. To solve for dz/dt, the value of z must be determined at the given points x=20 and y=30, which can be calculated from the original equation. This method ensures an accurate calculation of how fast z is increasing.
Runaway
Messages
48
Reaction score
0
1/z = 1/x + 1/y
If x is increasing at 4 units/s and y is increasing at 6 units/s, how fast is z increasing when x=20 and y=30?

My question is how do I find an equation for dz/dt? Does it equal dx/dt + dy/dt?
 
Physics news on Phys.org
You need to apply something like this


d/dt(f(x)) = (dx/dt)*d/dx(f(x)) = (dx/dt)f'(x)

Similar for f(y).
 
Or just differentiate implicitly with respect to t. On the left side you would have
d/dt(1/z) = d/dz(1/z) * dz/dt = -1/z2 * dz/dt.

dz/dt won't be equal to dx/dt + dy/dt. The relationship between x and y is nonlinear, so you won't get dx/dt + dy/dt when you differentiate the right side.
 
Mark44 said:
Or just differentiate implicitly with respect to t. On the left side you would have
d/dt(1/z) = d/dz(1/z) * dz/dt = -1/z2 * dz/dt.

dz/dt won't be equal to dx/dt + dy/dt. The relationship between x and y is nonlinear, so you won't get dx/dt + dy/dt when you differentiate the right side.

To continue on with what Mark44 said: \frac{dz}{dt}\frac{-1}{z^2} = \frac{dx}{dt}\frac{-1}{x^2} + \frac{dy}{dt}\frac{-1}{y^2} You have 2 unknowns, one is dz/dt which is what you are looking for and the other is z itself at that time. How would you find z? Hint: It's in your question!
 
Question: A clock's minute hand has length 4 and its hour hand has length 3. What is the distance between the tips at the moment when it is increasing most rapidly?(Putnam Exam Question) Answer: Making assumption that both the hands moves at constant angular velocities, the answer is ## \sqrt{7} .## But don't you think this assumption is somewhat doubtful and wrong?

Similar threads

Replies
2
Views
2K
  • · Replies 8 ·
Replies
8
Views
1K
  • · Replies 2 ·
Replies
2
Views
1K
Replies
1
Views
1K
  • · Replies 2 ·
Replies
2
Views
2K
Replies
12
Views
2K
Replies
3
Views
2K
  • · Replies 2 ·
Replies
2
Views
1K
Replies
12
Views
2K
Replies
4
Views
2K